Chapter 6 1 Which The Following Statements Correct You

subject Type Homework Help
subject Pages 14
subject Words 143
subject Authors Eugene F. Brigham, Michael C. Ehrhardt

Unlock document.

This document is partially blurred.
Unlock all pages and 1 million more documents.
Get Access
page-pf1
CHAPTER 6RISK AND RETURN
TRUE/FALSE
1. The tighter the probability distribution of its expected future returns, the greater the risk of a given
investment as measured by its standard deviation.
2. Risk-averse investors require higher rates of return on investments whose returns are highly uncertain,
and most investors are risk averse.
3. When adding a randomly chosen new stock to an existing portfolio, the higher (or more positive) the
degree of correlation between the new stock and stocks already in the portfolio, the less the additional
stock will reduce the portfolio's risk.
4. Diversification will normally reduce the riskiness of a portfolio of stocks.
5. In portfolio analysis, we often use ex post (historical) returns and standard deviations, despite the fact
that we are really interested in ex ante (future) data.
6. The realized return on a stock portfolio is the weighted average of the expected returns on the stocks in
the portfolio.
7. Market risk refers to the tendency of a stock to move with the general stock market. A stock with
above-average market risk will tend to be more volatile than an average stock, and its beta will be
greater than 1.0.
page-pf2
8. An individual stock's diversifiable risk, which is measured by its beta, can be lowered by adding more
stocks to the portfolio in which the stock is held.
9. Managers should under no conditions take actions that increase their firm's risk relative to the market,
regardless of how much those actions would increase the firm's expected rate of return.
10. One key conclusion of the Capital Asset Pricing Model is that the value of an asset should be
measured by considering both the risk and the expected return of the asset, assuming that the asset is
held in a well-diversified portfolio. The risk of the asset held in isolation is not relevant under the
CAPM.
11. According to the Capital Asset Pricing Model, investors are primarily concerned with portfolio risk,
not the risks of individual stocks held in isolation. Thus, the relevant risk of a stock is the stock's
contribution to the riskiness of a well-diversified portfolio.
12. If investors become less averse to risk, the slope of the Security Market Line (SML) will increase.
13. If a stock's expected return as seen by the marginal investor exceeds this investor's required return,
then the investor will buy the stock until its price has risen enough to bring the expected return down
to equal the required return.
14. If a stock's market price exceeds its intrinsic value as seen by the marginal investor, then the investor
will sell the stock until its price has fallen down to the level of the investor's estimate of the intrinsic
value.
page-pf3
15. For a stock to be in equilibrium, two conditions are necessary: (1) The stock's market price must equal
its intrinsic value as seen by the marginal investor and (2) the expected return as seen by the marginal
investor must equal this investor's required return.
16. Two conditions are used to determine whether or not a stock is in equilibrium: (1) Does the stock's
market price equal its intrinsic value as seen by the marginal investor, and (2) does the expected return
on the stock as seen by the marginal investor equal this investor's required return? If either of these
conditions, but not necessarily both, holds, then the stock is said to be in equilibrium.
17. Variance is a measure of the variability of returns, and since it involves squaring the deviation of each
actual return from the expected return, it is always larger than its square root, its standard deviation.
18. "Risk aversion" implies that investors require higher expected returns on riskier than on less risky
securities.
19. If investors are risk averse and hold only one stock, we can conclude that the required rate of return on
a stock whose standard deviation is 0.21 will be greater than the required return on a stock whose
standard deviation is 0.10. However, if stocks are held in portfolios, it is possible that the required
return could be higher on the stock with the low standard deviation.
page-pf4
20. Someone who is risk averse has a general dislike for risk and a preference for certainty. If risk aversion
exists in the market, then investors in general are willing to accept somewhat lower returns on less
risky securities. Different investors have different degrees of risk aversion, and the end result is that
investors with greater risk aversion tend to hold securities with lower risk (and therefore a lower
expected return) than investors who have more tolerance for risk.
21. A stock's beta measures its diversifiable risk relative to the diversifiable risks of other firms.
22. A stock's beta is more relevant as a measure of risk to an investor who holds only one stock than to an
investor who holds a well-diversified portfolio.
23. If the returns of two firms are negatively correlated, then one of them must have a negative beta.
24. A stock with a beta equal to 1.0 has zero systematic (or market) risk.
25. It is possible for a firm to have a positive beta, even if the correlation between its returns and those of
another firm is negative.
26. Portfolio A has but one security, while Portfolio B has 100 securities. Because of diversification
effects, we would expect Portfolio B to have the lower risk. However, it is possible for Portfolio A to
be less risky.
page-pf5
27. Portfolio A has but one stock, while Portfolio B consists of all stocks that trade in the market, each
held in proportion to its market value. Because of its diversification, Portfolio B will by definition be
riskless.
28. A portfolio's risk is measured by the weighted average of the standard deviations of the securities in
the portfolio. It is this aspect of portfolios that allows investors to combine stocks and thus reduce the
riskiness of their portfolios.
29. The distributions of rates of return for Companies AA and BB are given below:
State of the
Probability of
Economy
This State Occurring
AA
BB
Boom
0.2
30%
10%
Normal
0.6
10%
5%
Recession
0.2
5%
50%
We can conclude from the above information that any rational, risk-averse investor would be better off
adding Security AA to a well-diversified portfolio over Security BB.
30. Even if the correlation between the returns on two securities is +1.0, if the securities are combined in
the correct proportions, the resulting 2-asset portfolio will have less risk than either security held
alone.
31. Bad managerial judgments or unforeseen negative events that happen to a firm are defined as
"company-specific," or "unsystematic," events, and their effects on investment risk can in theory be
diversified away.
page-pf6
32. We would generally find that the beta of a single security is more stable over time than the beta of a
diversified portfolio.
33. We would almost always find that the beta of a diversified portfolio is less stable over time than the
beta of a single security.
34. If an investor buys enough stocks, he or she can, through diversification, eliminate all of the market
risk inherent in owning stocks, but as a general rule it will not be possible to eliminate all diversifiable
risk.
35. The CAPM is built on historic conditions, although in most cases we use expected future data in
applying it. Because betas used in the CAPM are calculated using expected future data, they are not
subject to changes in future volatility. This is one of the strengths of the CAPM.
36. Under the CAPM, the required rate of return on a firm's common stock is determined only by the
firm's market risk. If its market risk is known, and if that risk is expected to remain constant, then
analysts have all the information they need to calculate the firm's required rate of return.
37. A firm can change its beta through managerial decisions, including capital budgeting and capital
structure decisions.
38. Any change in its beta is likely to affect the required rate of return on a stock, which implies that a
change in beta will likely have an impact on the stock's price, other things held constant.
page-pf7
39. The slope of the SML is determined by the value of beta.
40. The slope of the SML is determined by investors' aversion to risk. The greater the average investor's
risk aversion, the steeper the SML.
41. If you plotted the returns of a company against those of the market and found that the slope of your
line was negative, the CAPM would indicate that the required rate of return on the stock should be less
than the risk-free rate for a well-diversified investor, assuming that the observed relationship is
expected to continue in the future.
42. If you plotted the returns on a given stock against those of the market, and if you found that the slope
of the regression line was negative, the CAPM would indicate that the required rate of return on the
stock should be greater than the risk-free rate for a well-diversified investor, assuming that the
observed relationship is expected to continue into the future.
43. The Y-axis intercept of the SML represents the required return of a portfolio with a beta of zero, which
is the risk-free rate.
44. The SML relates required returns to firms' systematic (or market) risk. The slope and intercept of this
line can be influenced by a manager's actions.
45. The Y-axis intercept of the SML indicates the required return on an individual asset whenever the
realized return on an average (b = 1) stock is zero.
page-pf8
46. If the price of money (e.g., interest rates and equity capital costs) increases due to an increase in
anticipated inflation, the risk-free rate will also increase. If there is no change in investors' risk
aversion, then the market risk premium (rM rRF) will remain constant. Also, if there is no change in
stocks' betas, then the required rate of return on each stock as measured by the CAPM will increase by
the same amount as the increase in expected inflation.
47. Since the market return represents the expected return on an average stock, the market return reflects a
certain amount of risk. As a result, there exists a market risk premium, which is the amount over and
above the risk-free rate, that is required to compensate stock investors for assuming an average amount
of risk.
48. Assume that two investors each hold a portfolio, and that portfolio is their only asset. Investor A's
portfolio has a beta of minus 2.0, while Investor B's portfolio has a beta of plus 2.0. Assuming that the
unsystematic risks of the stocks in the two portfolios are the same, then the two investors face the same
amount of risk. However, the holders of either portfolio could lower their risks, and by exactly the
same amount, by adding some "normal" stocks with beta = 1.0.
49. The CAPM is a multi-period model that takes account of differences in securities' maturities, and it can
be used to determine the required rate of return for any given level of systematic risk.
MULTIPLE CHOICE
50. If markets are in equilibrium, which of the following conditions will exist?
page-pf9
Each stock's expected return should equal its required return as seen by the marginal
investor.
All stocks should have the same expected return as seen by the marginal investor.
The expected and required returns on stocks and bonds should be equal.
All stocks should have the same realized return during the coming year.
Each stock's expected return should equal its realized return as seen by the marginal
investor.
51. You are considering investing in one of the these three stocks:
Stock
Standard Deviation
Beta
A
20%
0.59
B
10%
0.61
C
12%
1.29
If you are a strict risk minimizer, you would choose Stock ____ if it is to be held in isolation and Stock
____ if it is to be held as part of a well-diversified portfolio.
A; B.
B; A.
C; A.
C; B.
A; A.
52. Your friend is considering adding one additional stock to a 3-stock portfolio, to form a 4-stock
portfolio. She is highly risk averse and has asked for your advice. The three stocks currently held all
have b = 1.0, and they are perfectly positively correlated with the market. Potential new Stocks A and
B both have expected returns of 15%, are in equilibrium, and are equally correlated with the market,
with r = 0.75. However, Stock A's standard deviation of returns is 12% versus 8% for Stock B. Which
stock should this investor add to his or her portfolio, or does the choice not matter?
Stock A.
Stock B.
Neither A nor B, as neither has a return sufficient to compensate for risk.
Add A, since its beta must be lower.
Either A or B, i.e., the investor should be indifferent between the two.
page-pfa
53. Which of the following is NOT a potential problem when estimating and using betas, i.e., which
statement is FALSE?
Sometimes, during a period when the company is undergoing a change such as toward
more leverage or riskier assets, the calculated beta will be drastically different from the
"true" or "expected future" beta.
The beta of an "average stock," or "the market," can change over time, sometimes
drastically.
Sometimes the past data used to calculate beta do not reflect the likely risk of the firm for
the future because conditions have changed.
All of the statements above are true.
The fact that a security or project may not have a past history that can be used as the basis
for calculating beta.
54. Stock A's beta is 1.7 and Stock B's beta is 0.7. Which of the following statements must be true about
these securities? (Assume market equilibrium.)
Stock B must be a more desirable addition to a portfolio than A.
Stock A must be a more desirable addition to a portfolio than B.
The expected return on Stock A should be greater than that on B.
The expected return on Stock B should be greater than that on A.
When held in isolation, Stock A has more risk than Stock B.
55. Which of the following statements is CORRECT?
If you found a stock with a zero historical beta and held it as the only stock in your
portfolio, you would by definition have a riskless portfolio.
The beta coefficient of a stock is normally found by regressing past returns on a stock
against past market returns. One could also construct a scatter diagram of returns on the
stock versus those on the market, estimate the slope of the line of best fit, and use it as
beta. However, this historical beta may differ from the beta that exists in the future.
The beta of a portfolio of stocks is always larger than the betas of any of the individual
stocks.
It is theoretically possible for a stock to have a beta of 1.0. If a stock did have a beta of
1.0, then, at least in theory, its required rate of return would be equal to the risk-free
(default-free) rate of return, rRF.
The beta of a portfolio of stocks is always smaller than the betas of any of the individual
stocks.
page-pfb
56. Which of the following statements is CORRECT?
Suppose the returns on two stocks are negatively correlated. One has a beta of 1.2 as
determined in a regression analysis using data for the last 5 years, while the other has a
beta of 0.6. The returns on the stock with the negative beta must have been negatively
correlated with returns on most other stocks during that 5-year period.
Suppose you are managing a stock portfolio, and you have information that leads you to
believe the stock market is likely to be very strong in the immediate future. That is, you
are convinced that the market is about to rise sharply. You should sell your high-beta
stocks and buy low-beta stocks in order to take advantage of the expected market move.
You think that investor sentiment is about to change, and investors are about to become
more risk averse. This suggests that you should re-balance your portfolio to include more
high-beta stocks.
If the market risk premium remains constant, but the risk-free rate declines, then the
required returns on low-beta stocks will rise while those on high-beta stocks will decline.
Paid-in-Full Inc. is in the business of collecting past-due accounts for other companies,
i.e., it is a collection agency. Paid-in-Full's revenues, profits, and stock price tend to rise
during recessions. This suggests that Paid-in-Full Inc.'s beta should be quite high, say 2.0,
because it does so much better than most other companies when the economy is weak.
57. Which of the following statements is CORRECT?
Logically, it is easier to estimate the betas associated with capital budgeting projects than
the betas associated with stocks, especially if the projects are closely associated with
research and development activities.
The beta of an "average stock," which is also "the market beta," can change over time,
sometimes drastically.
If a newly issued stock does not have a past history that can be used for calculating beta,
then we should always estimate that its beta will turn out to be 1.0. This is especially true
if the company finances with more debt than the average firm.
During a period when a company is undergoing a change such as increasing its use of
leverage or taking on riskier projects, the calculated historical beta may be drastically
different from the beta that will exist in the future.
If a company with a high beta merges with a low-beta company, the best estimate of the
new merged company's beta is 1.0.
58. Stock A's beta is 1.7 and Stock B's beta is 0.7. Which of the following statements must be true,
assuming the CAPM is correct.
In equilibrium, the expected return on Stock B will be greater than that on Stock A.
When held in isolation, Stock A has more risk than Stock B.
Stock B would be a more desirable addition to a portfolio than A.
page-pfc
In equilibrium, the expected return on Stock A will be greater than that on B.
Stock A would be a more desirable addition to a portfolio then Stock B.
59. Stock X has a beta of 0.7 and Stock Y has a beta of 1.7. Which of the following statements must be
true, according to the CAPM?
Stock Y's realized return during the coming year will be higher than Stock X's return.
If the expected rate of inflation increases but the market risk premium is unchanged, the
required returns on the two stocks should increase by the same amount.
Stock Y's return has a higher standard deviation than Stock X.
If the market risk premium declines, but the risk-free rate is unchanged, Stock X will have
a larger decline in its required return than will Stock Y.
If you invest $50,000 in Stock X and $50,000 in Stock Y, your 2-stock portfolio would
have a beta significantly lower than 1.0, provided the returns on the two stocks are not
perfectly correlated.
60. Consider the following average annual returns for Stocks A and B and the Market. Which of the
possible answers best describes the historical betas for A and B?
Years
Market
Stock A
Stock B
1
0.03
0.16
0.05
2
0.05
0.20
0.05
3
0.01
0.18
0.05
4
0.10
0.25
0.05
5
0.06
0.14
0.05
bA > +1; bB = 0.
bA = 0; bB = 1.
bA < 0; bB = 0.
bA < 1; bB = 1.
bA > 0; bB = 1.
61.
page-pfd
Which of the following statements is CORRECT?
The higher the correlation between the stocks in a portfolio, the lower the risk inherent in
the portfolio.
An investor can eliminate almost all risk if he or she holds a very large and well
diversified portfolio of stocks.
Once a portfolio has about 40 stocks, adding additional stocks will not reduce its risk by
even a small amount.
An investor can eliminate almost all diversifiable risk if he or she holds a very large, well-
diversified portfolio of stocks.
An investor can eliminate almost all market risk if he or she holds a very large and well
diversified portfolio of stocks.
62. Which of the following statements is CORRECT?
If you were restricted to investing in publicly traded common stocks, yet you wanted to
minimize the riskiness of your portfolio as measured by its beta, then according to the
CAPM theory you should invest an equal amount of money in each stock in the market.
That is, if there were 10,000 traded stocks in the world, the least risky possible portfolio
would include some shares of each one.
If you formed a portfolio that consisted of all stocks with betas less than 1.0, which is
about half of all stocks, the portfolio would itself have a beta coefficient that is equal to
the weighted average beta of the stocks in the portfolio, and that portfolio would have less
risk than a portfolio that consisted of all stocks in the market.
Market risk can be eliminated by forming a large portfolio, and if some Treasury bonds
are held in the portfolio, the portfolio can be made to be completely riskless.
A portfolio that consists of all stocks in the market would have a required return that is
equal to the riskless rate.
If you add enough randomly selected stocks to a portfolio, you can completely eliminate
all of the market risk from the portfolio.
63. Recession, inflation, and high interest rates are economic events that are best characterized as being
company-specific risk factors that can be diversified away.
among the factors that are responsible for market risk.
risks that are beyond the control of investors and thus should not be considered by security
analysts or portfolio managers.
irrelevant except to governmental authorities like the Federal Reserve.
systematic risk factors that can be diversified away.
64. Which of the following statements is CORRECT?
page-pfe
If an investor buys enough stocks, he or she can, through diversification, eliminate all of
the diversifiable risk inherent in owning stocks. Therefore, if a portfolio contained all
publicly traded stocks, it would be essentially riskless.
The required return on a firm's common stock is, in theory, determined solely by its
market risk. If the market risk is known, and if that risk is expected to remain constant,
then no other information is required to specify the firm's required return.
Portfolio diversification reduces the variability of returns (as measured by the standard
deviation) of each individual stock held in a portfolio.
A security's beta measures its non-diversifiable, or market, risk relative to that of an
average stock.
A stock's beta is less relevant as a measure of risk to an investor with a well-diversified
portfolio than to an investor who holds only that one stock.
65. Which of the following statements is CORRECT?
Diversifiable risk can be reduced by forming a large portfolio, but normally even highly-
diversified portfolios are subject to market (or systematic) risk.
A large portfolio of randomly selected stocks will have a standard deviation of returns that
is greater than the standard deviation of a 1-stock portfolio if that one stock has a beta less
than 1.0.
A large portfolio of stocks whose betas are greater than 1.0 will have less market risk than
a single stock with a beta = 0.8.
If you add enough randomly selected stocks to a portfolio, you can completely eliminate
all of the market risk from the portfolio.
A large portfolio of randomly selected stocks will always have a standard deviation of
returns that is less than the standard deviation of a portfolio with fewer stocks, regardless
of how the stocks in the smaller portfolio are selected.
66. Which of the following statements is CORRECT?
A portfolio that consists of 40 stocks that are not highly correlated with "the market" will
probably be less risky than a portfolio of 40 stocks that are highly correlated with the
market, assuming the stocks all have the same standard deviations.
A two-stock portfolio will always have a lower beta than a one-stock portfolio.
If portfolios are formed by randomly selecting stocks, a 10-stock portfolio will always
have a lower beta than a one-stock portfolio.
A stock with an above-average standard deviation must also have an above-average beta.
A two-stock portfolio will always have a lower standard deviation than a one-stock
portfolio.
page-pff
67. Consider the following information for three stocks, A, B, and C. The stocks' returns are positively but
not perfectly positively correlated with one another, i.e., the correlations are all between 0 and 1.
Expected
Standard
Stock
Return
Deviation
Beta
A
10%
20%
1.0
B
10%
10%
1.0
C
12%
12%
1.4
Portfolio AB has half of its funds invested in Stock A and half in Stock B. Portfolio ABC has one third
of its funds invested in each of the three stocks. The risk-free rate is 5%, and the market is in
equilibrium, so required returns equal expected returns. Which of the following statements is
CORRECT?
Portfolio AB's coefficient of variation is greater than 2.0.
Portfolio AB's required return is greater than the required return on Stock A.
Portfolio ABC's expected return is 10.66667%.
Portfolio ABC has a standard deviation of 20%.
Portfolio AB has a standard deviation of 20%.
68. Which of the following statements is CORRECT?
A portfolio with a large number of randomly selected stocks would have more market risk
than a single stock that has a beta of 0.5, assuming that the stock's beta was correctly
calculated and is stable.
If a stock has a negative beta, its expected return must be negative.
A portfolio with a large number of randomly selected stocks would have less market risk
than a single stock that has a beta of 0.5.
According to the CAPM, stocks with higher standard deviations of returns must also have
higher expected returns.
If the returns on two stocks are perfectly positively correlated (i.e., the correlation
coefficient is +1.0) and these stocks have identical standard deviations, an equally
weighted portfolio of the two stocks will have a standard deviation that is less than that of
the individual stocks.
69. Which of the following is most likely to be true for a portfolio of 40 randomly selected stocks?
The riskiness of the portfolio is the same as the riskiness of each stock if it was held in
isolation.
The beta of the portfolio is less than the average of the betas of the individual stocks.
The beta of the portfolio is equal to the average of the betas of the individual stocks.
The beta of the portfolio is larger than the average of the betas of the individual stocks.
The riskiness of the portfolio is greater than the riskiness of each of the stocks if each was
held in isolation.
page-pf10
70. If you randomly select stocks and add them to your portfolio, which of the following statements best
describes what you should expect?
Adding more such stocks will increase the portfolio's expected rate of return.
Adding more such stocks will reduce the portfolio's beta coefficient and thus its systematic
risk.
Adding more such stocks will have no effect on the portfolio's risk.
Adding more such stocks will reduce the portfolio's market risk but not its unsystematic
risk.
Adding more such stocks will reduce the portfolio's unsystematic, or diversifiable, risk.
71. Charlie and Lucinda each have $50,000 invested in stock portfolios. Charlie's has a beta of 1.2, an
expected return of 10.8%, and a standard deviation of 25%. Lucinda's has a beta of 0.8, an expected
return of 9.2%, and a standard deviation that is also 25%. The correlation coefficient, r, between
Charlie's and Lucinda's portfolios is zero. If Charlie and Lucinda marry and combine their portfolios,
which of the following best describes their combined $100,000 portfolio?
The combined portfolio's beta will be equal to a simple weighted average of the betas of
the two individual portfolios, 1.0; its expected return will be equal to a simple weighted
average of the expected returns of the two individual portfolios, 10.0%; and its standard
deviation will be less than the simple average of the two portfolios' standard deviations,
25%.
The combined portfolio's expected return will be greater than the simple weighted average
of the expected returns of the two individual portfolios, 10.0%.
The combined portfolio's standard deviation will be greater than the simple average of the
two portfolios' standard deviations, 25%.
The combined portfolio's standard deviation will be equal to a simple average of the two
portfolios' standard deviations, 25%.
The combined portfolio's expected return will be less than the simple weighted average of
the expected returns of the two individual portfolios, 10.0%.
72. The two stocks in your portfolio, X and Y, have independent returns, so the correlation between them,
rXY is zero. Your portfolio consists of $50,000 invested in Stock X and $50,000 invested in Stock Y.
Both stocks have an expected return of 15%, betas of 1.6, and standard deviations of 30%. Which of
the following statements best describes the characteristics of your 2-stock portfolio?
Your portfolio has a standard deviation less than 30%, and its beta is greater than 1.6.
Your portfolio has a beta equal to 1.6, and its expected return is 15%.
Your portfolio has a beta greater than 1.6, and its expected return is greater than 15%.
Your portfolio has a standard deviation greater than 30% and a beta equal to 1.6.
Your portfolio has a standard deviation of 30%, and its expected return is 15%.
page-pf11
73. Which of the following is most likely to occur as you add randomly selected stocks to your portfolio,
which currently consists of 3 average stocks?
The expected return of your portfolio is likely to decline.
The diversifiable risk will remain the same, but the market risk will likely decline.
Both the diversifiable risk and the market risk of your portfolio are likely to decline.
The total risk of your portfolio should decline, and as a result, the expected rate of return
on the portfolio should also decline.
The diversifiable risk of your portfolio will likely decline, but the expected market risk
should not change.
74. Ann has a portfolio of 20 average stocks, and Tom has a portfolio of 2 average stocks. Assuming the
market is in equilibrium, which of the following statements is CORRECT?
The required return on Ann's portfolio will be lower than that on Tom's portfolio because
Ann's portfolio will have less total risk.
Tom's portfolio will have more diversifiable risk, the same market risk, and thus more
total risk than Ann's portfolio, but the required (and expected) returns will be the same on
both portfolios.
If the two portfolios have the same beta, their required returns will be the same, but Ann's
portfolio will have less market risk than Tom's.
The expected return on Jane's portfolio must be lower than the expected return on Dick's
portfolio because Jane is more diversified.
Ann's portfolio will have less diversifiable risk and also less market risk than Tom's
portfolio.
75. Stocks A and B are quite similar: Each has an expected return of 12%, a beta of 1.2, and a standard
deviation of 25%. The returns on the two stocks have a correlation of 0.6. Portfolio P has 50% in Stock
A and 50% in Stock B. Which of the following statements is CORRECT?
Portfolio P has a standard deviation that is greater than 25%.
Portfolio P has an expected return that is less than 12%.
Portfolio P has a standard deviation that is less than 25%.
Portfolio P has a beta that is less than 1.2.
Portfolio P has a beta that is greater than 1.2.
page-pf12
76. Stocks A, B, and C are similar in some respects: Each has an expected return of 10% and a standard
deviation of 25%. Stocks A and B have returns that are independent of one another; i.e., their
correlation coefficient, r, equals zero. Stocks A and C have returns that are negatively correlated with
one another; i.e., r is less than 0. Portfolio AB is a portfolio with half of its money invested in Stock A
and half in Stock B. Portfolio AC is a portfolio with half of its money invested in Stock A and half
invested in Stock C. Which of the following statements is CORRECT?
Portfolio AC has an expected return that is greater than 25%.
Portfolio AB has a standard deviation that is greater than 25%.
Portfolio AB has a standard deviation that is equal to 25%.
Portfolio AC has a standard deviation that is less than 25%.
Portfolio AC has an expected return that is less than 10%.
77. Stocks A and B each have an expected return of 15%, a standard deviation of 20%, and a beta of 1.2.
The returns on the two stocks have a correlation coefficient of +0.6. Your portfolio consists of 50% A
and 50% B. Which of the following statements is CORRECT?
The portfolio's expected return is 15%.
The portfolio's standard deviation is greater than 20%.
The portfolio's beta is greater than 1.2.
The portfolio's standard deviation is 20%.
The portfolio's beta is less than 1.2.
78. Stock A has a beta of 0.8, Stock B has a beta of 1.0, and Stock C has a beta of 1.2. Portfolio P has 1/3
of its value invested in each stock. Each stock has a standard deviation of 25%, and their returns are
independent of one another, i.e., the correlation coefficients between each pair of stocks is zero.
Assuming the market is in equilibrium, which of the following statements is CORRECT?
Portfolio P's expected return is equal to the expected return on Stock A.
Portfolio P's expected return is less than the expected return on Stock B.
Portfolio P's expected return is equal to the expected return on Stock B.
Portfolio P's expected return is greater than the expected return on Stock C.
Portfolio P's expected return is greater than the expected return on Stock B.
79. In a portfolio of three randomly selected stocks, which of the following could NOT be true; i.e., which
statement is false?
The standard deviation of the portfolio is greater than the standard deviation of one or two
of the stocks.
The beta of the portfolio is lower than the lowest of the three betas.
The beta of the portfolio is equal to one of the three stock's betas.
The beta of the portfolio is equal to 1.
page-pf13
The standard deviation of the portfolio is less than the standard deviation of each of the
stocks if they were held in isolation.
80. Stock A has a beta = 0.8, while Stock B has a beta = 1.6. Which of the following statements is
CORRECT?
If the marginal investor becomes more risk averse, the required return on Stock B will
increase by more than the required return on Stock A.
An equally weighted portfolio of Stocks A and B will have a beta lower than 1.2.
If the marginal investor becomes more risk averse, the required return on Stock A will
increase by more than the required return on Stock B.
If the risk-free rate increases but the market risk premium remains constant, the required
return on Stock A will increase by more than that on Stock B.
Stock B's required return is double that of Stock A's.
81. Stock A has an expected return of 12%, a beta of 1.2, and a standard deviation of 20%. Stock B also
has a beta of 1.2, but its expected return is 10% and its standard deviation is 15%. Portfolio AB has
$300,000 invested in Stock A and $100,000 invested in Stock B. The correlation between the two
stocks' returns is zero (that is, rA,B = 0). Which of the following statements is CORRECT?
The stocks are not in equilibrium based on the CAPM; if A is valued correctly, then B is
overvalued.
The stocks are not in equilibrium based on the CAPM; if A is valued correctly, then B is
undervalued.
Portfolio AB's expected return is 11.0%.
Portfolio AB's beta is less than 1.2.
Portfolio AB's standard deviation is 17.5%.
82. You have a portfolio P that consists of 50% Stock X and 50% Stock Y. Stock X has a beta of 0.7 and
Stock Y has a beta of 1.3. The standard deviation of each stock's returns is 20%. The stocks' returns are
independent of each other, i.e., the correlation coefficient, r, between them is zero. Given this
information, which of the following statements is CORRECT?
The required return on Portfolio P is equal to the market risk premium (rM rRF).
Portfolio P has a beta of 0.7.
Portfolio P has a beta of 1.0 and a required return that is equal to the riskless rate, rRF.
Portfolio P has the same required return as the market (rM).
Portfolio P has a standard deviation of 20%.
page-pf14
83. Which of the following statements is CORRECT? (Assume that the risk-free rate is a constant.)
The effect of a change in the market risk premium depends on the slope of the yield curve.
If the market risk premium increases by 1%, then the required return on all stocks will rise
by 1%.
If the market risk premium increases by 1%, then the required return will increase by 1%
for a stock that has a beta of 1.0.
The effect of a change in the market risk premium depends on the level of the risk-free
rate.
If the market risk premium increases by 1%, then the required return will increase for
stocks that have a beta greater than 1.0, but it will decrease for stocks that have a beta less
than 1.0.
84. In historical data, we see that investments with the highest average annual returns also tend to have the
highest standard deviations of annual returns. This observation supports the notion that there is a
positive correlation between risk and return. Which of the following answers correctly ranks
investments from highest to lowest risk (and return), where the security with the highest risk is shown
first, the one with the lowest risk last?
Large-company stocks, small-company stocks, long-term corporate bonds, U.S. Treasury
bills, long-term government bonds.
Small-company stocks, large-company stocks, long-term corporate bonds, long-term
government bonds, U.S. Treasury bills.
U.S. Treasury bills, long-term government bonds, long-term corporate bonds, small-
company stocks, large-company stocks.
Large-company stocks, small-company stocks, long-term corporate bonds, long-term
government bonds, U.S. Treasury bills.
Small-company stocks, long-term corporate bonds, large-company stocks, long-term
government bonds, U.S. Treasury bills.
85. Suppose that during the coming year, the risk free rate, rRF, is expected to remain the same, while the
market risk premium (rM rRF), is expected to fall. Given this forecast, which of the following
statements is CORRECT?
The required return on all stocks will remain unchanged.
The required return will fall for all stocks, but it will fall more for stocks with higher betas.
The required return for all stocks will fall by the same amount.
The required return will fall for all stocks, but it will fall less for stocks with higher betas.
The required return will increase for stocks with a beta less than 1.0 and will decrease for
stocks with a beta greater than 1.0.

Trusted by Thousands of
Students

Here are what students say about us.

Copyright ©2022 All rights reserved. | CoursePaper is not sponsored or endorsed by any college or university.